Physics Competitions Vol 13

Download as pdf or txt
Download as pdf or txt
You are on page 1of 60

Vol 13 no 2 2011

Vol 14 no 1 2012

ISSN 1389 - 6458


Physics

• The Armchair Space


Traveler
• Iran in WWMD 2007-2009
• Oscillators Everywhere
• Physics Education
and PYPT
• Operating Temperature
Journal
and Heat Capacity of a
of the
Light Bulb Filament
• A few good Orbits World
Federation
of

Physics

Competitions
Physics Competitions

Contents Page

WFPhC, Executive Committee, Advisory Board and Award Committee........... 2

Editorial................................................................................................ 6

Award, Reports on Competitions and Articles

WFPhC award 2010……………………………………………………………… 8


H. Jordens

The Armchair Space Traveler……………………………………………………. 10


J. Boyd, D. Hinson, R. J. Palmaccio

Iran in WWMD 2007-2009……………………………………………………. 19


D. Izadi

Oscillators Everywhere……………………………………………………………. 26
J. Boyd, R. J. Palmaccio, W. Yong

Physics Education and PYPT……………………………………………………. 35


D. Izadi, H. Ghaednia, R. M. Namin, S. Zargham, Z. Karimi

Reviewed Articles

Operating Temperatur and Heat Capacity of a Light Bulb Filament…………. 42


A. Boeltzig, S. Petersen

A few good Orbits……………………………………………………………….. 50


Chiraag Juvekar, Mehul Jain and Aniket Sule
Physics Competitions Vol 13 No 2 2011 & Vol.14 No 1 2012

Physics Competitions

W
PhF C
EXECUTIVE COMMITTEE
President: Helmuth MAYR / Austria
retired physics and physics didactics teacher / Vienna
federal coordinator of the Austrian Physics Olympiad
[email protected]
Vice-Presidents: Gunnar FRIEGE / Germany
Institute of Mathematics and Physics Education
Leibniz University Hanover
Welfengarten 1; D-30167 Hannover
[email protected]

Hendra KWEE / Indonesia


General secretary of APhO
Surya Institute
[email protected]

Secretary: Stefan PETERSEN / Germany


Leibnitz Institute for Science and Mathematics Education
at the University of Kiel
Olshausenstraße 61; D-24118 Kiel
[email protected]

Treasurer: Ingibjörg HARALDSDOTTIR / Iceland


Kopavogur Gymnasium
Digranesvegi
IS-200 Kopavogur
[email protected]
members:
Andrzej KOTLICKI / Canada
University of British Columbia
Department of Physics and Astronomy
6224 Agricultural Road
Vancouver B.C.
Canada V6T 1Z1
[email protected]

2
Physics Competitions Vol 13 No 2 2011 & Vol.14 No 1 2012
Paul PSHENICHKAL / Ukraine
Representative of ICYS
[email protected]

Natalya KAZACHKOVA / Ukraine


[email protected]

THE AWARDS COMMITTEE


Masno GINTING / Indonesia
Komplek Puspiptek Blok II H no. 13
Tangerang Selatan-Banten
15310
[email protected]

Andrei LAVRINENKO / Denmark


Technical University of Denmark
DK-2800 Lyngby
[email protected]
Juan LEON / Spain
IFF/CSIC
Serrano 113-B
ES-28006 Madrid
http//quinfog.iff.csic.es
[email protected]
Jan MOSTOWSKI / POLAND
Polish Academy of Science
Warszawa
[email protected]
Monika RAHARTI / Indonesia
Surya Institute
[email protected]

THE ADVISORY BOARD


Giuliana CAVAGGIONI / Italy
Italian Association for the Teaching of Physics
[email protected]
Andrei LAVRINENKO / Denmark
Technical University of Denmark
DK-2800 Lyngby
[email protected]
Andrzej NADOLNY / Poland
Polish Academy of Science
Warszawa
[email protected]

3
Physics Competitions Vol 13 No 2 2011 & Vol.14 No 1 2012

REGIONAL REPRESENTATIVES
Dr. Alan Alliosn
Australia
Mr. Michael A. Cotter
Ireland

Dr. Gagik V. Grigoryan,


Armenia

Prof. MSc. Ozimar Pereira


Brazil

EDITOR-IN-CHIEF
Prof. Dr. Helmuth Mayr
retired physics and physics didactics teacher / Vienna
federal coordinator of the Austrian Physics Olympiad
[email protected]

TECHNICAL SUPPORT
Prof. Dr. Gunnar Friege / Silke Reichel / Phillip Niemann
Institut für Didaktik der Mathematik und Physik,
Welfengarten 1, 30167 Hannover, Germany

EDITORIAL BOARD
Prof. Dr. Masno Ginting
Komplek Puspiptek Blok II H no. 13, Serpong-Banten
Indonesia 15310

Drs. Hans Jorden


Department of Physics, University of Groningen, Nijenborgh 4,
9747 AG Groningen, The Netherlands

Prof. Dr. Gunnar Friege


Institut für Didaktik der Mathematik und Physik,
Welfengarten 1, 30167 Hannover, Germany

4
Physics Competitions Vol 13 No 2 2011 & Vol.14 No 1 2012

The aims of the Federation are:

1. To promote excellence in, and research associated with, physics education


through the use of school physics competitions;

2. To promote meetings and conferences where persons interested in physics


contests can exchange and develop ideas of use in their countries;

3. To provide opportunities for the exchanging of information for physics


education through published material, notably through the Journal of the
Federation;

4. To recognize through the WFPhC Awards system persons who have made
notable contributions to physics education through physical challenge around
the world;

5. To organize assistance provided by countries with developed system for


competitions for countries attempting to develop competitions;

6. To promote physics and to encourage young physicists.

5
Physics Competitions Vol 13 No 2 2011 & Vol.14 No 1 2012

EDITORIAL

In April 2012 the 5th Congress took place in a holiday resort near Nijmegen in the
Netherlands. We had chosen to have this Congress just before the International
Conference for Young Scientists (ICYS) so that the participants would have the
chance to attend both, the Congress and the Conference.
The theme of the Congress was “Interdisciplinarity” and we will report on it in the
next Journal.

Following our statutes, there were elections during the Congress. The result was a
new situation:
Hans Jordens from the Netherlands, who had acted as WFPhC-president from the
beginning of the Federation in the year 2002, had decided to retire from his position
and wanted to be “just” an “ordinary member” of the Federation in future.
Dear Hans Jordens,
We have to say “Thank you very much !”. You played an important role in the
Federation. It was you who founded it, together with Waldemar Gorzkowski, and
after Waldemar passed away very unexpectedly in 2007, you followed Waldemar in
his job as IPHO-president in 2008. In all those years you did a great job in our
Federation !

At the same time Zsuzsanna Rajkovits from Hungary, up to this very date acting as
vice-president, wanted to stop her work in the Federation.
Dear Zsuzsanna,
You did an important job, very often more in the background, and we have to say
“Thank you very much!” for all your activities.
Zsuzsanna Rajkovits was chosen from the Awards Committee to get the prize of the
year 2010. It was given to her during the 4-th Congress, and later in this journal you
can find an article about that event.

Matti Rajamäki from Finland, being the head of the Award Committee, also wanted
to terminate his activities.
Dear Matti,
Once again we have to say “Thank you so much!” for your activities, both in this
Committee and in the Executive Committee.

Yohanes Surya from Indonesia also wanted to stop his activities in the Executive
Committee. That´s why we have to say, once again: “Dear Yohannes! Thank you so
much for all the work you did for our Federation!”

Jan Mostowski from Poland chose not to continue his activities in the Executive
Committee and changed over to the Awards Committee.

Dear Jan! Thank you very much for all your commitment !

6
Physics Competitions Vol 13 No 2 2011 & Vol.14 No 1 2012

The Awards Committee decided to give the Award-2012 to Maija Ahtee from
Finland. She was head of the Finnish team from the very beginning of Finland´s

participation in the IPHO up to 1999. From this very year up to 2007 she acted as
secretary of the International Physics Olympiad. As the former IPHO-president,
Waldemar Gorzkowski, passed away during the IPHO-2007, Maija Ahtee was acting
president up to 2008, when the new president, Hans Jordens, was elected. From
that date up to 2009 Maija Ahtee worked as secretary again. Then she stopped her
engagements in the IPHO.
Maija Ahtee took an important role in the IPHO, very often more in the background
than on stage. It is planned to hand the prize over to Maija during the IPHO-2012 in
Estonia. She will attend this IPHO as a special guest.

Hence there is a new situation in our Federation with new people in several
functions, which one can read at the beginning of this journal.
I am sure that the new crew will continue all the activities of the Federation in the
sense of the founders. I very well remember all the discussions we had in 2002 in
Bali, when the Federation was founded and will do my best to act in this sense.

Helmuth Mayr
June 2012
[email protected]

7
Physics Competitions Vol 13 No 2 2011 & Vol.14 No 1 2012

The WFPhC- award 2010 Dr. Zsuzsanna Rajkovits


During the congress of the Federation it was a great pleasure for me to hand the
WFPhC-award 2010 to Dr. Zsuzsanna Rajkovits from the Öetvös Lorand University
of Budapest, Hungary, for her outstanding achievements in the education of physics.
For me the pleasure was double because I think Dr.Rajkovits, who largely contributed
and still contributes to the aims of the Federation, not only deserves the prize, but for
me it was also because of personal reasons that I was happy about her to receive the
award. More than two decades ago we met for the first time when we were visiting
the museum of the Kremlin as part of an excursion of the International Young
Physicists’ Tournament. Soon it became clear that our ideas about education and
most of all our mutual fascination for physics, created a natural bond between us
both. That bond has remained unchanged ever since. From very close I know how
much effort and enthusiasm Zsuzsa puts into her work for physics education. Her
contributions in various competitions as well as in presentations and articles are
numerous.

In 1994 Zsuzsa created, together with colleague Dr. L. Markovich, a new competition
called the International Conference of Young Scientists. This competition aims to
give the floor to youngsters to present the results of their research in a similar way as
‘real’ researchers do during international conferences, as is stated in article 1.1.:
“The International Conference of Young Scientists (ICYS) is essentially an individual
competition on scientific research and presentations carried out by school students

8
Physics Competitions Vol 13 No 2 2011 & Vol.14 No 1 2012
which are evaluated by the International Jury.” Very often the participants show their
fascination for the subject they have researched by presentations that can easily meet
the standard of professionals. The way in which they discuss about different topics of
their presentation not seldom show profound knowledge and enthusiasm. An
important aspect of the conference is that the participants are stimulated to meet and
make contacts, and in some cases friendships are created for the rest of their lives.
The ICYS is a very successful and truly international competition in which now some
200 participants from all over the world participate.

When Zsuzsa was asked to spend her efforts for the World Federation of Physics
Competitions, she did not hesitate for a second and agreed to become vice-president,
a position which she occupies till today. As a member of the Executive Committee
she co-organized the past congresses and ever since than she gave presentations
herself during these congresses. Many articles for the journal of the Federation:
“Physics Competitions” appeared from her hand.

The decision by the award committee of the WFPhC on who should receive the
award in 2010 was unanimous. This decision was welcomed with a warm applause by
all the participants of the 4th Congress of the Federation in 2010 in Baske Ostarija,
Croatia.

Hans Jordens

9
Physics Competitions Vol 13 No 2 2011 & Vol.14 No 1 2012

The Armchair Space Traveler

Jim Boyd1, Don Hinson2 & Richard J. Palmaccio3

1MathematicsDepartment, St. Christopher’s School, Virginia, USA


274Asheridge Rd, CHESHAM HP5 2PY, England, UK
3Mathematics Department, St. Sebastian’s School, Needham, MA USA

Abstract
This is an article presents a vicarious adventure into space through the examination
of various problems in physics. The first takes the armchair traveler to the moon.
Upon arrival there, the second problem concerns a study of elements in a sample
taken from the moon’s surface. The next move is a journey to Jupiter as the third
problem studies the “slingshot” effect whereby the gravity of Jupiter is used to
accelerate a spaceship past that planet. Finally, our armchair journey takes a cosmic
leap with the final problem dealing with the use of a fusion reaction to gain more
acceleration for a trip to the stars.

1. Introduction

The Age of Space Travel was inaugurated in 1957 when Russian engineers placed
their first Sputnik in orbit around the earth. Shortly thereafter, they sent a dog
around the world aboard their second Sputnik. Thus the first space passenger to
orbit the Earth was canine rather than human. Then, in 1961, the Russian
cosmonaut Yuri Gagarin was launched into a single orbit, thus qualifying as the first
human space traveler (of whom we are aware). The first space venture other than
“around the World and back again” occurred in 1965 when the unmanned Russian
craft Venera 3 crashed onto the surface of Venus. This flight was followed in 1966
by the soft landing of the American Surveyor 1 on the Moon.

True space travel in which a human being visited a body in space other than the
Earth began in 1969 with the flight of the American Apollo 11 which carried Neil
Armstrong, Buzz Aldrin, and Michael Collins to the Moon. While Collins remained
behind circling the Moon in the command module, the lunar landing craft carried
Armstrong and Aldrin to the lunar surface where they safely disembarked to take
their triumphal walks. The world watched the space adventure and the safe return of
the three astronauts on its television screens.

Just as the expeditions of the Nineteenth Century and early Twentieth Century
explorers who traveled to the unknown regions of planet Earth stimulated interest in
both real and vicarious terrestrial travel, so the voyages of the first cosmonauts and
astronauts have inspired actual and armchair space travel. Armchair space travel is
the safer and more comfortable of the two varieties, but with paper, pencil, and
calculator, we can investigate and solve problems that might confront real space
travelers while remaining in the comfort of our own studies. Thus armchair space
travel can possess a reality beyond the reading of descriptions of journeys to the
source of the Nile, or treks across Antarctica to the South Pole.
10
Physics Competitions Vol 13 No 2 2011 & Vol.14 No 1 2012

2. The Problems

The first problem has three parts.


Problem 1. (Flying to the Moon)

A. What is the escape velocity for a rocket launched from the surface of the
Earth?
B. What is the minimum energy per kilogram required to deliver a spacecraft to
the Moon from the surface of the Earth?
C. How much kinetic energy per kilogram must be dissipated (by retro-rockets) in
order that the spacecraft make a soft landing on the lunar surface?

Before proceeding with our solutions, we list the numerical values in S.I. units of the
physical constants that will appear in our calculations:

g = gravitational acceleration at the Earth’s surface = 9.81


G = universal gravitational constant = 6.672 x 10-11
R(0) = distance between the centers of the Earth and Moon =
3.844 x 108
R(E) = radius of the Earth = 6.37 x 106
R(M) = radius of the Moon = 1.738 x 106
M = mass of the Earth = 5.974 x 1024
m = mass of the Moon = 0.735 x 1024

Solution.
A. Let us assume that all of the rocket’s kinetic energy T is delivered to it at a single
shot at the instant of lift-off and that T is sufficient to carry the rocket beyond the
Earth’s gravitational field (i.e. to “infinity”). Furthermore, let us assume that air
resistance does not act on the rocket as it travels through the Earth’s atmosphere.
Then

T     
0


where µ and v represent the mass of the rocket after its fuel has been spent, and the
initial velocity of the rocket, respectively.

11
Physics Competitions Vol 13 No 2 2011 & Vol.14 No 1 2012

If v is taken to be the escape velocity, then


 GMµ
  v   (1)
 R E

Equation (1) states that all of the kinetic energy delivered to the rocket at lift-off is
converted to gravitational potential energy. Since   
, equation (1) may be

rewritten as    2 . Solving this equation for v and using the values given above
for the relevant constants, we find the escape velocity to be   11.19 #$&%.

B. Since the spacecraft does not have to leave the earth’s gravitational field, but only
travel to the Moon, and since the Moon’s gravitational pull on the spacecraft will be
helpful in the flight, the kinetic energy required to send the spacecraft from the
surface of the Earth to the surface of the Moon is less than that associated with the
escape velocity.
To simplify our thinking, les us imagine that the spacecraft travels along the line of
centers of the Earth and the Moon. We then note that there is a point, say P, on this
line at which the gravitational attractions by the Earth and the Moon precisely cancel
one another out. Now let the distance of P from the center of the Earth be denoted
by r. Since the gravitational forces by the Earth and the Moon on the spacecraft are
(
of equal size but in opposite directions at P, we may write that '   )*' which
)
implies that '
 1 + ,$⁄-  1.111. Thus /  3.46 x 108 m.

The work performed on 1 kilogram of mass in carrying it from the surface of the
Earth to point P is given by
' *
01  2 
 3  )*3
 45   '
+ 
 )*'
 )* 
 (2)

Evaluating the right hand side of equation (2) by substituting the value of r as found
above and the values of the physical constants listed at the beginning of our solution
yields the kinetic energy converted to gravitational potential energy as 1 kilogram is
lifted from the surface of the Earth to point P. Any kinetic energy in excess of T’ will
carry the kilogram to P and, thereafter, the kilogram will plummet to the lunar
surface under the gravitational attraction of the Moon. The value of T’ may be taken
to be the minimum value of initial kinetic energy per kilogram which will send the
spacecraft to the Moon. We find that T’ = 6.146 x 107 J/kg.
The initial velocity associated with this kinetic energy is given by v 1  √20 1  11.09
km/s which is, as argued above, slightly less than the escape velocity calculated in
part A of our solution.

C. The distance of the “zero gravity” point P from the center of the Moon is given by
r’ = R(0) – r 3.84 x 107 m. The work required to lift 1 kilogram from the surface of the
Moon to P is
' *
0 11  2 
 3   45   +    (3)
)*3 '  )*' )* 

Evaluating T’’ as we evaluated T’, we find that T’’ = 2.584 x 107 J/kg. This is also
the kinetic energy per kilogram that should be dissipated if the spacecraft is to touch
down on the Moon’s surface with zero velocity. We note in conclusion that the
12
Physics Competitions Vol 13 No 2 2011 & Vol.14 No 1 2012
minimum lift-off velocity that will send the kilogram from the Moon’s surface back to
P is √20 11  2.273 km/s.

Once we reach the Moon, we ought to do some science. One reason for going to
the Moon might be to examine the materials on its surface. So, our next problem
which comes in two parts suggests a project that we might pursue.

Problem 2. (Doing Science on the Moon).

A. A sample taken from the surface contained 4.0 grams of potassium of which
0.01% was in the form of the isotope which decays into argon with a half-
life of 1.2 x 109 years. When the sample was heated in a vacuum, 2.4 cubic
centimeters of argon at a temperature of 20o C and a pressure of 101.3 kPa
were released. Calculate from these data a minimum age for the Moon.
Assume that all of the argon in the sample was produced by the decay of
potassium.
B. Alpha particles were used to bombard a small region of the lunar surface near
the site from which the sample of part A above was taken. The purpose of the
bombardment was to identify the chemical elements in the Moon’s soil. In one
experiment, the maximum kinetic energy of the recoiling alpha particles was
37% of their incident kinetic energy. Identify the element with whose nuclei
the alpha particles collided.

13
Physics Competitions Vol 13 No 2 2011 & Vol.14 No 1 2012

Solution.

A. The 4 grams of potassium constitutes 0.1 mole which implies that there is 10-5 of
a mole of ;)
:9 . Sample calculations based on the Ideal Gas Law, Pv = nRT, yield the
number of moles of argon released:
101.3 < 10=  < 2.4 < 10*?   @ 8.31 < 293 B @  10*; $CDE
We can now use the equation of exponential decay
F G  F 0E *HI (4)
to compute the age of the Moon based on the assumptions that we have made. In
this equation, A(0) represents the amount of in the sample at t = 0 (the “instant”
at which the Moon was formed). In our problem, A(0) = 10-5 + 10-4 = 11 x 10-5
mole. The parameter λ can be calculated from the half-life 1.2 x 109 years or
radioactive potassium by noting that λ = (ln 2)/(1.2 x 109) = 5.776 x 10-10. Making
the appropriate substitutions into equation (4) and letting A( t ) = 10-5, we solve the
equation to find that t = 4.15 x 109 years. This value is the estimated age of the
Moon.

B. The maximum recoil energy will occur when an alpha particle suffers a head-on
collision with the nucleus of an atom under bombardment. In this case, the approach
and separation of the alpha particle and nucleus before and after collision will take
place on a fixed line. Furthermore, we shall assume that, before collision, the
nucleus was at rest.

We take the mass of the alpha particle to be 4 mass units and the mass of the
nucleus that we wish to identify to be M mass units. We denote the velocity of the
alpha particle before it collides with the stationary nucleus by v and the velocity of
the nucleus after the collision by u. The Law of Conservation of Energy implies that
  
 4   0.37  4  +  -J .

Simplification yields
L 
-  2.52 M . (5)

Since the kinetic energy of the alpha particle after the collision is 37% of the incident
kinetic energy, its recoil velocity must be √0.37 v = 0.6087 v. Then the Law of
Conservation of Momentum implies that
4v = Mu – 4(0.608)v .

14
Physics Competitions Vol 13 No 2 2011 & Vol.14 No 1 2012
This result in turn implies that
L
-  6.432 M. (6)

Solving equations (5) and (6) simultaneously yields M = 16.4 mass units. Thus we
may infer that the element with which the alpha particle collided was oxygen.

Maximizing the efficient use of fuel in space is of prime importance. Robinson


Crusoe’s isolation would be as nothing compared to being lost in space. All ways of
saving fuel ought to be investigated and exploited if possible. One concept is that of
the “slingshot” principle. The idea is that when a spacecraft is traveling in the
vicinity of a planet which is itself approaching the spacecraft (with respect to the rest
frame of the fixed stars), a transfer of energy can occur which results in an
increase in the speed of the spacecraft. Of course, the pull of the planet’s
gravitational field will slow the spacecraft down somewhat, but the velocity with
which the spacecraft departs may still exceed the velocity with which it approached
the planet.

A rough analogy may make the “slingshot” effect understandable to our armchair
traveler. Let us imagine the level swing of a massive bat as it moves to strike an
incoming ball. We take the ratio of the masses of the bat and ball to be comparable
to the ratio of the masses of the spacecraft and planet. That is, the mass of the bat
may be taken to be infinitely great. Let us assume that the collision between the bat
and ball is perfectly elastic. Then, in the rest frame of the bat, the ball will rebound
without loss of kinetic energy. The velocity v of the ball will be reversed in direction
by its collision with the bat, but the magnitude of the ball’s velocity will remain
constant. Returning to the rest frame fixed on the baseball diamond, we see an
increase in the magnitude of the velocity of the ball as its speed before the collision is
reduced by the speed of the bat while its speed after the collision is increased by the
same amount.

Thus the speed of the ball is increased by 2u where u is the speed of the bat. The
forces developed during the collision were contact forces. In the interaction between
the spacecraft and the planet, the forces are gravitational and contact does not occur
since a crash landing would be undesirable. However, the basic idea is the same for
the ball-bat and spacecraft-planet interactions.

15
Physics Competitions Vol 13 No 2 2011 & Vol.14 No 1 2012

Problem 3. (Getting a Lift).

A spaceship with a speed of v = 15 km/s approaches the planet Jupiter which is


moving toward it at a speed of u = 13 km/s with respect to the fixed stars. The
approach is oblique and at a distance so great that gravitational attractions may be
neglected. The path of the spacecraft makes an angle of 30 degrees with the
direction of the velocity of the oncoming planet. After being attracted and deflected
by the planet, the spacecraft emerges from Jupiter’s influence with its course altered
by 300 degrees as it heads away at the angle of 30 degrees on the other side of
Jupiter’s direction. Find the gain in speed acquired by the spacecraft during its
interaction with the planet.

Figure 1. Jupiter’s “Slingshot”.

The geometry of the problem is suggested by Figure 1.

Solution.

Resolve the initial velocity of the spaceship into components parallel and
perpendicular to the planet’s path. Then
v (perpendicular, 0) = 15 sin 30o = 7.5 and v (parallel, 0) = 15 cos 30o = 12.99 .
16
Physics Competitions Vol 13 No 2 2011 & Vol.14 No 1 2012
The component of velocity perpendicular to the planet’s path will be unaffected by
the interaction between the spaceship and the planet. By the argument advanced in
our analogy of the batted ball, the direction of the parallel component of velocity will
be reversed and its magnitude increased by 2(13) = 26. The components of velocity
as the spaceship heads away from Jupiter will be
v (perpendicular, 1) = 15 sin 30o = 7.5 and
v (parallel, 1) = -15 cos 30o – 26 = -38.99 .
The gain in speed will be √7.5 + 38.99  √7.5 + 12.99  24.71 #$⁄%. Thus Jupiter’s
“slingshot” increases the spaceship’s speed by almost 25 km/s without the
expenditure of any fuel at all!

The armchair imagination is completely free. So here is a really far out idea for
saving fuel. Might not an interstellar spaceship collect protons in space as fuel? The
protons would be fed into a fusion reactor to generate kinetic energy to accelerate
the ship. Those interested in collecting protons should know that their density in
space is n = 0.1 proton per cubic centimeter which is 105 protons per cubic meter.

Problem 4. (Accelerating the Spaceship).

How large a scoop would have to be designed to collect enough protons to develop
an acceleration a = 10 m/s2 for a 3500 tonne ship if f = 0.01 of the energy of the
fusion reactor is converted to kinetic energy of the spaceship? Assume that 100% of
the kinetic energy produced is converted to thrust. For protons to be captured they
must incident on the surface of the scoop mechanism.
We need to recall that a mass of 1 tonne is M = 1000g, the mass of a proton is m =
1.7 x 10-27 kg, and the speed of light is c = 3 x 108 m/s in order to complete our
calculations.

Solution.

Let us suppose that the velocity of the spaceship is v and that trailing behind the
spaceship is the scoop which has a cross-sectional surface area A presented
perpendicular to the direction of the spaceship’s travel. The volume of space swept
for protons per second by the scoop will be Av and the mass of protons collected per
second will be nmAv = 1.7 x 10-22 Av kg/s. Using Einstein’s relationship for the
17
Physics Competitions Vol 13 No 2 2011 & Vol.14 No 1 2012

energy equivalence of mass, we find that the energy collected is 1.7 x 10-22 Avc2 =
1.53 x 10-5 Av J/s. This result represents an input of power. Of this input,
f (1.53 x 10-5 Av) = 1.53 x 10-7Av J/s is converted to thrust.

The power required to develop the thrust to give the ship an acceleration of a = 10
m/s2 is 3500Mav = 3.5 x 107 v J/s. Equating the two values of power just obtained,
we find that
1.53 x 10-7 Av = 3.5 x 107 v
implying that A = 2.29 x 1014 square meters, an enormous surface area indeed. If
the scoop were circular, its radius would be , 2.29 < 10; ⁄O  8.538 x 106 meters =
8537 km. Although the idea was an interesting one, such a scoop would be
completely impractical.

Our armchair astronaut was a bit disappointed that his idea turned out to be
unrealistic. However, he was not one to give up so easily. He had read about
“wormholes” that might connect distant parts of the universe. Perhaps he could find
an entrance. And then, in just the briefest of moments before it closed, he could

18
Physics Competitions Vol 13 No 2 2011 & Vol.14 No 1 2012

Iran in WWMD 2007-2009

Dina Izadi

Ariaian Young Innovative Minds Institute (AYIMI), Tehran, Iran


http://www.ayimi.org, [email protected]

1. Introduction
World Water Monitoring Day (WWMD) is an international education and outreach
program by coordination of Water Monitoring Day, the Water Environment
Federation (WEF) and the International Water Association (IWA) that builds public
awareness and involvement in protecting water resources around the world by
engaging citizens to conduct basic monitoring of their local water bodies. World
Water Monitoring Day activities takes place from March through December each year
and December 31 served as the deadline for reporting. All the participants sampled
their local lakes, streams, rivers, ponds, estuaries and other Water bodies for four key
water quality indicators: dissolved oxygen (DO), pH, temperature and turbidity. Some
groups also monitored for the presence of certain macro invertebrates such as
dragonflies, mayflies and scuds. Samples were taken in a range of settings—
agricultural, commercial, residential and industrial—on six continents. A total of
122,599 participants monitored sites worldwide, which represents a 67% increase
over 2008. In Iran we have started this program since 2007. Each year we send
some educational kits to the universities and schools that are willing to participate in
these practical activities. These kits are sent by the WWMD organizers. We organized
the 1st scientific workshop in 2009 and one of the topics was WWMD. Participants
learned how to use the kits and how to do in situ experiments.

2. Monitoring Four Rivers in Iran for WWMD™, 2007


Sikhoran, Roudan, Karaj and Jagerood Rivers.
In this project students from Bandar Abbas Azad University (Flora
Mohamadizade,Marzie Haj Keramadini, Abbas Barkhordari, Shadi Khatami, Maria
Mohamadizade, Parviz Tavakoli) and 10 students (Atena Shirdastian, Shiva
Mahdaviseresht, Noshad Khosravi, Golnoosh Azarbakhsh, Yasaman Mehrabi, Orkide
Olang, Mahsa Kave, Negin Amini, Farimah Ramezanpoor, Niloofar Keramati) from
Aboureihan educational complex took part.
Hormozgan province covering an area of 68,476 km2 is located southern Iran, north
of the Persian Gulf. The main part of the province is covered by mountainous
regions. The Zagross mountain range extends from northeast to southeast and
terminates to lime and sandy hills, highlands, as well as coastal lowlands, parallel to
the Persian Gulf and Oman Sea. Roudan and Sikhoran are ancient geographical
regions of this province. The measurements in Karaj River were carried out in a part
of this river in Gachsar to Chaloos road (about 6 kilometers after Karaj) on a sunny
day and Jagerood measurements near Latia Dam (Fig. 1).

19
Physics Competitions Vol 13 No 2 2011 & Vol.14 No 1 2012

Figure 1. In situ experiments in Iran, 2007

3. Results in 2007
The following figures represent the results for 2007 based upon the four WWMD™
water quality parameters in Iran. These are listed by country and do not constitute a
completely thorough and accurate portrayal of the health of the world’s water.
Credible water quality sampling requires using standard quality assurance protocols
and is conducted with trained volunteer monitoring groups and professionals around
the world (Table 1).
Table 1. Results in Iran , 2007
Country Total Total Average Average Average Average
Sites Participants DO Temperature pH Turbidity
(PPM) (°C) (JTU)
Iran 6 46 4.00 15.72 6.78 13.33

4. Numerical Summary 2007


Forty-three countries participated in World Water Monitoring Day™ 2007 and
46,117 participants reported data from a total of 3,544 sites. Test Kits Distributed
were about 10,636 and monitored sites were 3,544 and Total Participants were
46,117. The following figures represent the results of WWMD™ 2007 for the
continents of North America, South America, Europe, Asia, Africa and Australia
based upon the four WWMD™ water quality parameters (table 2).

20
Physics Competitions Vol 13 No 2 2011 & Vol.14 No 1 2012
Table 2. Water quality parameters 2007
(http://www.worldwatermonitoringday.org/2007)

The in situ experiments have been done in different countries to get more details
about water quality in the world (Fig. 2).

Fig. 2 Average DO, PH, Temperature and Turbidity,


2007(http://www.worldwatermonitoringday.org/2007)

5. Monitoring different Rivers in Iran for WWMD™ ,2009


In 2009, over 120,000 people in 81 countries monitored their local waterways. The
1st workshop on Nov. 20, 2009 was organized by the Ariaian Young Innovative
Minds Institute (AYIMI) with cooperation of Iranian Society of Marine Science and
Technology (ISMST) and the Scientific Societies of Amirkabir University of
Technology (AUT) and students registered to receive the kits.
Students from Rahe Roshd education complex in Tehran (Jeiran Joorabchi, Tina
Aghajooni, Sadaf Farhoodi, Diba Bagheri, Golsa Saadati, Parinaz Sadoughi, Negar
Shariat, Ava Mohammadi, Ariyaneh Abedinnejad, Mahrouya Nikoufar, Kiana Abbasi,
Sogand Shariatmadari, Mahsa Abbaspour, DonyaMazloumi, Saba Sendani and
teamleader Tayari) as a team work and a student from Amin School in Tabriz (Farid
Mehri) sent their reports to the AYIMI and we provided all details for WWMD. Their
measurements were from the samples of Darake Local River, Karaj lake dam and
Darband Local River. Darband and Darake are some kind of recreational centers to
visit and Karaj’s dam is one of the most important water reservoirs of Tehran. They
started sampling on 14th and 15th of December. At both of the rivers were no kinds

21
Physics Competitions Vol 13 No 2 2011 & Vol.14 No 1 2012

of extra pollutions in the samples of water. Karaj’s dam is a restricted area so the
water was clean and with no kind of serious pollutions.
Table 3. Results in Iran, 2009
Country Participants Sites DO pH Water Turbidity (JTU)
(PPM) Temperature (°C)
Iran 56 23 4.56 7.31 5.79 17.25
By sampling these local waters they found that the water resources and the
ingredients are one of the most important elements in their daily lives and should
make effort protecting them from any kind of damages and pollutions and they have
to know that living without healthy water is impossible.

6. Watershed domain Karaj


Small water area in Karaj of approximately 1115 square kilometers in north-eastern
city of Karaj was sampled. The analysis done on that project within Hydrometric
Station melting snow 80 percent, which is variable due to having field, 41 percent to
6 percent of the maximum domain of Karaj during various boards of at least
mountainous conditions and type of winter precipitation that is often the snow. River
Darakeh: These areas in the north-west of Tehran and the median area of Khoshkeh
and Velenjak. Domain area is of 2500 hectares of mountain boards and the Altitude
3876 meters above sea level. The first kilometer of the River Valley, which is
Darakeh approximate width of 10 m and depth of this part of the average is 4
meters.
Table 4. Water quality parameters in Darband, Darake and Karaj, 2009
Location Date Time Air Water Tur Do pH Saturation
Temp Temp (JTU) (ppm)
(˚C) (˚C)
Darband 2009/12/14 8:26 0 2 0 8 7 58%
creek A.M
Darake 2009/12/14 10:58 2 2 0 8 8 58%
creek A.M
Karaj dam 2009/12/14 4:30 4 6 40 4 7.5 31%
lake P.M
Darband 2009/12/15 8:13 2 3 0 8 7 61%
A.M
Darake 2009/12/15 10:30 2 4 0 8 7.5 61%
P.M
Karaj 2009/12/15 15:12 4 6 40 8 7.5 64%
P.M

7. Kandowan Area for sampling


Kandowan is a popular town in Eastern Azerbaijan of Iran. There is a river with name
of Kandowan Chaei. Also there is a pipe that pumps the water from Kandowan
spring that has Mineral Water. Farid Mehri tested both of them individualy. After

22
Physics Competitions Vol 13 No 2 2011 & Vol.14 No 1 2012
hiking in Sahand mountain he went to a lake on top of the mountain and tested the
water. In Basmenj city there is a river which he tested it too.

Table 5. . Water quality parameters in Azerbaijan, 2009


Location Date Coordinates Air Water Tur. Do pH
Temp Temp (JTU) (ppm)
(˚C) (˚C)
Kandowan 2009/12/25 37.81346.3113 3 2.5 50.000 4 8
River
Kandowan 2009/12/25 37.73146.3345 3 4 0 3 7
Spring
(Mineral
Water)
The lake on 2009/12/25 37.726746.3465 -2 2.3 0 2 6
top of the
Sahand
mountain
Basmenj 2009/12/25 37.985946.4921 8 6 30.000 0 6
river
Jeghateh 2009/12/26 36.971246.1149 14 10.6 30.000 4 8
chaei
Zarrine 2009/12/26 36.98246.1091 14 9.3 6.660 8 8

Fig. 3. In situ experiments in Iran, 2009

8. Numerical Summary 2009


Over 120000 people participated in World Water Monitoring Day™ 2009. Test Kits
Distributed were about 16000 and monitored sites were 8000. The following figures
represent the results of WWMD™ 2009 for the continents of North America, South
America, Europe, Asia, Africa and Australia based upon the four WWMD™ water
quality parameters (table 6)

23
Physics Competitions Vol 13 No 2 2011 & Vol.14 No 1 2012

Table 6. Water quality parameters,


2009(http://www.worldwatermonitoringday.org/2009)

The average PH, DO, temperature and turbidity were measured to compare water
quality (Fig. 4).

Fig. 4.Average DO, PH, Temperature and Turbidity, 2009


(http://www.worldwatermonitoringday.org/2009)
9. Conclusions
The actual monitoring shows some human activities like washing and swimming along
the streams causes pollutions and people need healthy water to drink. Surveys at
various sites included measurements for temperature, pH, dissolved oxygen and
turbidity are done by monitoring equipments supplied by the World Water Monitoring
Day program. The testing water exists in the world help us to know what is accessible
to us, and why it is important to conserve and protect our water.
Acknowledgements
It is nice to thank students from Bandar Abbas Azad University, Aboureihan and
Rahe Roshd educational complex in Tehran and from Amin School in Azarbaijan
which took part in this project in 2007 and 2009.
References: http://www.worldwatermonitoringday.org/2007
http://www.worldwatermonitoringday.org/2009, http://www.ayimi.org

24
Physics Competitions Vol 13 No 2 2011 & Vol.14 No 1 2012

Oscillators Everywhere

Jim Boyd1, Richard J. Palmaccio2 & Willie Yong3

1Mathematics Department, St. Christopher’s School, Virginia, USA


2Mathematics Department, St. Sebastian’s School, Needham, MA USA
3 SCT Publishing, Singapore

Abstract
We present a collection of problems about simple harmonic motion. These problems
target students who are preparing for the Asian Physics Olympiads, various national
Physics Olympiads, and the International Physics Olympiads. We hope that students will
find the problems interesting and useful.

1. Introduction

Physicists have given us a model of the world filled with vibration at all scales in the
universe from the smallest, grainiest quantum levels to the vast objects of study in
astrophysics which demand a unification of general relativity and quantum mechanics.

The domain of classical physics is the “reasonable” world that we experience directly
every day. The objects treated by classical physics are neither too small nor too large
and their velocities are comfortably less than the speed of light. That domain might
be called the “Goldilocks World” because everything therein is “just right.” The
ubiquity of vibratory motion holds throughout the “Goldilocks World” in which we
live, and the first and often most sensible approximation of classical vibratory motion
is simple harmonic oscillation, the projection of uniform circular motion onto a
diameter. We suspect that more ink has been expended in writing “F = ma” and
variants thereof than in writing any other equation of classical physics. A strong case
could be argued that “m(d2x/dt2) = -k x” accounts for a considerable portion of that
ink. The bobbing of a cork in water, the beating of the wings of birds and insects,
the swinging of a pendulum, and even the tremors of the ground that we stand upon
in the shock and after-shock of earthquakes are examples of vibrations that we have
or may have observed in daily life. Since physicists seem to prefer “oscillation” as a
word to “vibration”, we feel that the title of our article is justified.

But let us stop for just a moment on our way to our first problem in order to
comment on the vibratory motion induced by an earthquake. We urge readers to
turn to the Internet to find information about the Yasaka Pagoda in Kyoto, Japan.
The first version of the structure was built before 600 CE. From time to time during
its history, the pagoda caught fire, burned and was rebuilt. The present structure
dates from 1440. The design of the Pagoda which the citizens of Kyoto and visitors
to that city see today serves as a model for “earthquake proof” construction. The
five stories of the Pagoda are attached independently to a tall, central, vertical pillar.
The response of the Pagoda to the energy that it absorbs with the shock of an
earthquake is that each of the five levels moves freely about the central pillar with a
damped oscillation as the energy is transmitted through the pillar to the ground and
reabsorbed there.

25
Physics Competitions Vol 13 No 2 2011 & Vol.14 No 1 2012

The central supporting pillar was first incorporated into the design during a
reconstruction a little before 940. Thus builders of long ago took advantage of their
understanding of vibratory motion.

2. The Problems

Now, on to the problems! We shall present four problems. We shall give solutions
to the first three but simply state the fourth, leaving its solution as a challenge and
source of pleasure for our readers.

Problem 1. You are sitting in your garden sipping orange juice on a nice summer
afternoon. Let us imagine a bee in search of just the right flower. Having spotted a
nectar-laden blossom, the bee hovers above before settling down to gather the sweet,
raw material for the hive’s production of honey. As the bee hovers in midair, it is the
motion of the bee’s wings that produces the familiar buzz which we hear. Estimate
the frequency of the bee’s buzz.

Solution. Let the surface area of each of each of the bee’s wings be denoted by A.
Let the maximum amplitude of the flapping motion of the bee’s wings be denoted by
zo. Let us denote the angular frequency of flapping by ω sec-1 and finally, let us
denote the density of the air by ρ and the mass of the bee by m.

To help our thinking along, we draw a figure to suggest the bee hovering above a
flower.

A
P)
Horizontal Line
P)

Figure 1. The Bee and the Flower

26
Physics Competitions Vol 13 No 2 2011 & Vol.14 No 1 2012
We must compute the upward force exerted by the air as it supports the weight mg of
the bee. To do so, we compute the force exerted on the air by the bee during the
downward stroke of its wings. The opposite reaction force by the air over the
surfaces of the wings supports the weight of the bee.
The volume of air moved by each complete downward stroke of both wings is
2(2zo)(kA) where k is a geometric constant depending on the shape of the region of
space swept out by one complete downward stroke of each wing. The first factor of
2 is needed because there are two wings in motion. The second factor of 2 is needed
because the tip of each wing travels twice the amplitude with each flap from top to
bottom.

Sensible arguments may be advanced that #  . In any event, we make that

assumption and take the volume of air displaced by a complete down stroke of both
wings to be 2zoA. Then the mass of the displaced air becomes
-QR'  2SFP) .
The assumption that the motion of the wings is simple harmonic is equivalent to
assuming that the displacement of each wing may be written as a function of time in
the form
T G  P) sin XG.
The acceleration of each wing is then
Y Z
 P) X sin XG.
YI 

The magnitude of the force exerted by the bee on the air below it may be
approximated by
Y Z
[$QR' YI  [  2SFP) X sin XG (1)

with 0 \ XG \ O since we claim that the bee is not affected by the air during the half-
period of the upward stroke if its wings.

By Newton’s Third Law of Motion, the magnitude of the force that supports the bee
during the down stroke will also be given by equation (1). Over the complete period
of the motion of the wings, the supporting force is
2SFP) X sin XG 4J/_@ G`E 4Ca@ %G/C#Ec
]^
0 4J/_@ G`E Jb %G/C#E
Next we compute the average value of the supporting force F over a complete period
d
. We find that.
e
d⁄e
2) 2SFP) X sin XG 4G 2SFP) X
]QLf   .
2O⁄X O
We must now make yet another assumption which is that zo is comparable to the
dimensions of the bee’s wings. We accomplish that by assuming that zo = √F so that
we may write that
2SF X
]QLf  .
O

27
Physics Competitions Vol 13 No 2 2011 & Vol.14 No 1 2012

Finally, we equate the average supporting force to the weight of the hovering bee.
We obtain
2SF X
 $
O
or
d(f
Xg .
hi

We conclude by computing the approximate angular frequency ω for reasonable


values of the parameters in our problem. Let m = 0.001 gm, A = 0.170 cm2,
ρ = 0.0013 gm/cm3, and g = 980 cm/sec2. We find that ω  200 sec-1 which
agrees well with observed buzz frequencies of 200 to 250 sec-1.

Problem 2. Here is a familiar problem. A wooden disc bobs at the surface of an


otherwise still pond. The dimensions of the pond are so great with respect to the
dimensions of the disc that the level of the water surface remains unchanged as the
motion of the disc proceeds. The buoyant force exerted by the water exceeds the
weight of the disc when the disc is at its greatest depth and is less than the weight of
the disc when it is at the top of its bobbing trajectory. When written, the equation of
motion of the disc is easily seen to be that of simple harmonic motion.

Let us now look at a more complicated problem. That problem involves a solid
wooden disc of radius a, thickness t, and uniform density ρwood which floats in a liquid
which is contained in a vessel having dimensions comparable to the dimensions of the
disc. When the disc is displaced from its equilibrium position, the resulting change in
the level of the liquid surface may not be ignored.

Let us say that the density of the liquid is ρliq and that the vessel containing the liquid
is a deep circular cylinder of radius R. We emphasize that we do not assume a << R.

Suppose that the disc is pushed down so that its horizontal upper surface is slightly
above the level of the liquid surface and that the disc is then released from rest.
Thereafter, the disc bobs up and down without damping by frictional effects while
remaining upright at all times.

Determine the angular frequency of the vertical oscillations of the disc.

Solution. Let us say that the volume of the liquid in the cylindrical vessel is Vo and
that the depth of the liquid would be ho if the disc were not floating and bobbing in
the liquid. Figure 2a

28
Physics Competitions Vol 13 No 2 2011 & Vol.14 No 1 2012

Figure 2. The Geometry of the Bobbing Disc.

shows the cylindrical vessel filled with the liquid to a depth of ho before the disc is
inserted. Figure 2b shows the vessel with the disc displacing a volume of liquid equal
to the volume of the part of the disc which is now below the higher surface level of
the liquid. We have drawn the Figure as though the central axes of the disc and the
cylindrical vessel coincide. As the disc bobs upward, the liquid level descends; as the
disc bobs downward, the liquid level rises. As shown in Figure 2b, the horizontal
surface of the liquid is x units above ho and the lower surface of the disc is y units
below ho.

We begin our computations by relating x and y. We note that the volume V of the
liquid is not changed by inserting the disc. We may write that V = πR2ho (see Figure
2a) and that
V = πOR2(ho + x) – πOa2(x + y) (see Figure 2b). Therefore,
OR2ho = πOR2(ho + x) – πOa2(x + y)
which implies that 0 = R2x – a2(x + y) or
Q j
5  *Q . (2)
Next, we analyze the vertical forces acting on the disc and write an equation of
motion. The varying upward buoyant force Fo exerted by the liquid on the disc is the
weight of the liquid displaced by the disc. That force is FB = g ρliq(x + y)πOa2 where g
is the acceleration due to gravity. The constant downward force acting on the disc is
its weight given by W = g ρwood t Oa2.

The magnitude of the unbalanced vertical force acting on the disc is


|]l  m|  nOo pSqRr 5 + s  StuuY Gvn.
The mass of the wood is ρwood πa2t. Since y is the displacement of the lower surface
Y j
of the disc from a fixed level, represents the acceleration of the disc in a fixed
YI 
frame of reference.

29
Physics Competitions Vol 13 No 2 2011 & Vol.14 No 1 2012

Since the direction of the unbalanced force is obviously opposite to the


displacement of the disc, we may now apply Newton’s Second Law of Motion to
write an equation of motion:
4s
StuuY Oo G  Oo pSqRr 5 + s  StuuY Gv.
4G 
Simplification yields
4s SqRr StuuY
  w5 + s  x y Gz.
4G  GStuuY SqRr
Recalling that equation (2) gives x as a function of y, we rewrite the equation of
motion as
4s SqRr o StuuY

  wx  
+ 1y s  x y Gz.
4G GStuuY   o SqRr
Further simplification yields
Y j fh{|}  Ih~€  *Q

YI 
  Ih   *Q  xs   h{|}
‚  y . (3)
~€

Finally, we make the change of variable


GStuuY    o
J sx yx y
SqRr 
and transform equation (3) into an equation for simple harmonic motion. We now
have
4J SqRr 
   ‚ x y J.
4G  GStuuY    o
The motion is simple harmonic in the variable u with y oscillating about its
Ih~€  *Q
equilibrium value of  ‚ 
. The desired angular frequency of the oscillation
h{|}
is

SqRr 
X  ƒ ‚x  y.
GStuuY   o

To get a sense of the numbers involved in our problem, we let the disc bob in water
in a cylindrical vessel or radius R = 8 cm. We suppose that the disc is made of white
oak and has radius a = 5 cm and thickness t = 4 cm. Water has density 1 gm/cm3
and the white oak used for the disc has the high density of 0.77 gm/cm3. We
assume that g = 980 cm/sec2. Evaluating ω for the values given, we find that the
e
angular frequency would be 22.85 sec-1 which implies a vibrational frequency of d =
3.64 sec-1.

Problem 3. A right circular cone with altitude h and volume V has a base of radius r.
The vertex angle is θ. The entire mass M of the cone is concentrated at its vertex.
The cone is turned so that its vertex points downward and it is then placed in a liquid
of density ρ > M/V so that it floats with its altitude aligned with the vertical direction.

30
Physics Competitions Vol 13 No 2 2011 & Vol.14 No 1 2012
The reader may imagine a light conical drinking cup floating in the liquid with a small
but heavy metal weight resting at its vertex as suggested by Figure 3.1. We assume
that, if the cone bobs up and down, the level of the liquid surface will not be affected
and that viscuous drag on the cone may be ignored. We also assume that there will
be no horizontal perturbations of the cone as it oscillates along the vertical direction.

We ask first for a differential equation of motion for the vertex of the cone as it bobs
up and down. Since this equation will turn out to be nonlinear, we also ask for a
linear approximation to this equation and for the corresponding simple harmonic
frequency for oscillations of very small amplitude.

Figure 3.1 The Right Circular Cone.

Solution. We begin by computing the equilibrium depth of the floating cone. We


denote the depth of the vertex below the liquid surface by y and the radius of the
circular cross section of the cone at the liquid surface by x. We show the geometry
of the bobbing cone in Figure 3.2.

Figure 3.2 The Geometry of the Bobbing Cone.

31
Physics Competitions Vol 13 No 2 2011 & Vol.14 No 1 2012

Let yo denote the depth of the vertex when the cone is floating at rest in equilibrium.
3 ‡ ‡
The radius x will have the corresponding value xo where j„  tan . Thus 5)  s) tan .
„  

At the equilibrium position, the weight of the liquid displaced by the cone must equal
the weight of the cone Mg where g is the acceleration due to gravity. Thus
‡
O5) s) OS Go@  s)= 

-  S 
3 3
which implies that
⁄=
=
s)  w ‰ z . (4)
dh IQˆ


The last calculations may be repeated in deriving the equations of motion. As the
cone bobs up and down, the upward buoyant force is given by
‡
OS Go@  
]l  Š ‹ s=
3

where y varies as opposed to remaining fixed at yo while the weight of the cone
keeps the constant value Mg. Taking the downward direction to be positive for y, the
equation of motion becomes
‰
Y j dhIQˆ fj Œ

- YI   -  =
(5)

Let us recall that in the calculations leading up to equation (4), we showed that
‰
dhIQˆ fj„Œ

-  . This recollection allows us to rewrite equation (5) as
=
 ‡ ‡
4  s OS Go@   = OS Go@  
-  =
s)  s  s)  s s) + su s + s  .
4G 3 3
In seeking a simple harmonic approximation of the bobbing cone, we take |yo – y|
to be very small so that s  s) . Thus our last equation becomes
‡ ‡

4  s OS Go@   OS Go@  s)

 s)  s 3s)  s)  s.
4G  3- -
Letting u  y  y) , we may write that
‰
Y M dhIQˆ fj„

 J.
YI 

32
Physics Competitions Vol 13 No 2 2011 & Vol.14 No 1 2012

The angular frequency for simple harmonic approximation is

‡ 
1 ƒOS Go@  s)
X .
2O -
Substituting the expression for yo given by equation (4) into the equation for ω and
simplifying the result yields

 
‡ Œ = ‡ ‘
9OSGo@ 9OS Go@
X   Š 
‹ Š 
‹.
- -

Then the frequency of vibration for the cone in its simple harmonic approximation is

= ‡ ‘
X 1 9OS Go@ 
’  Š ‹.
2O 2O -

Finally, let us imagine that the cone is bobbing up and down in water. To obtain a
feel for the motion for which we have given a mathematical description, let us assign
numbers to our symbols and determine the frequency f.

The density of water is ρ = 1 gm/cm3. Let θ = 30o, M = 200 grams, and g = 980
cm/sec2. We find that f  2.32 sec-1.

Problem 4. For our Readers


(1992 British Physics Olympiad)
A light pulley of radius R has half of its curved surface covered by a uniform strip of
metal sheet of total mass M as indicated in Figure 4. A cord is wound round the
pulley and one of its ends is attached to a mass m that hangs vertically.

Figure 4. The Pulley with the Strip of Metal and Mass m attached to the Cord.

33
Physics Competitions Vol 13 No 2 2011 & Vol.14 No 1 2012

Verify that the distance of the centre of mass X of the metal strip from the axis
O of the pulley is (2R/O).

Determine each of the following:


The equilibrium inclination, θ, of OX to the vertical for all values of m for
which equilibrium is possible.

The values of m for which equilibrium is not possible.

The period of oscillation of the system, for small angular displacements α from
the stable equilibrium orientation θ, using energy considerations, or the
equation of motion.

By means of displacement-time graphs for m, describe its possible modes of


motion.
˜
Note that for small α, cos θ + α  cos θ  α cos θ  
cos θ + ™

34
Physics Competitions Vol 13 No 2 2011 & Vol.14 No 1 2012

Physics Education and PYPT

Dina Izadi

Ariaian Young Innovative Minds Institute (AYIMI), Tehran, Iran


http://www.ayimi.org, [email protected]

Abstract
It has been recognized that physics learning and teaching should change from
traditional to research method and findings from Physics Education Research (PER)
had initiated a tremendous effort in improving physics teaching. New methods
indicate that student understanding and misconceptions from quantitative
interpretation research has built a pathway for curriculum development and effective
teaching of introductory physics. Active learning is a method which has caused to
increase student conceptual understanding. This encourages students to construct
their own knowledge. There is increasing evidence that many students are unable to
apply the physics that they have studied. For meaningful learning to occur, students
need more assistance than they can obtain through listening to lectures, reading the
textbook, and solving standard quantitative problems. Substitution the traditional
models of teaching and learning physics with new one will promote active learning.
Different national and international tournaments which are either valuable to students
or educators will improve their skills. Persian Young Physicists’ Tournament, PYPT,
is one of the programs which Iran has involved since 2007. This tournament provides
practice in interpreting various representations, e.g., formulas, graphs, diagrams, and
verbal descriptions. Students solve the problems by research which is one of the most
important parameters in physics education. PYPT workshops are another opportunity
for students who have been in previous PYPTs to educate other students and learn
how to be a teacher. In this case students will be trained to become future scientists.
Ariaian Young Innovative Minds Institute (AYIMI) is the organizer of PYPT in Iran
which holds this tournament with cooperation of universities and organizations each
year to select team for International Young Physicists’ Tournament (IYPT).

1. Introduction
Scientific laws, principles and concepts can be better understood and experienced by
the interactions of individuals solving problems in groups and in a laboratory style
and the correlation between algorithmic mathematical skills and problem solving in
physics.
During the last 15 years, in most countries, the popularity of physics among students
has been low and the enrolment has declined so different ways instead of traditional
physical science instruction will lead to improve physics education for all students.
In active learning, Studio Model will help students work in a group and do their
experiments to get the physics laws and teachers just help them with the
experiments, arranging discussion, and guiding students toward a correct conclusion.
Discovery Lab will provide an opportunity for students to carry out experiments with

35
Physics Competitions Vol 13 No 2 2011 & Vol.14 No 1 2012

mathematical expressions. They experiment to collect evidence and draw a


conclusion based on that evidence.
Most students like novices see physics more as isolated pieces of information
unrelated to the real world. But what we specifically should do students think like an
expert. They should learn physics by experiments and see the content of physics as
the concepts that describe nature and use in a wide variety of situations. Students
should be able to solve problems correctly by learning the useful concept not by
memorizing. So instructors can move students from mindless memorization to
understanding. If teachers try to teach physics in the context of everyday life
applications, students are more likely to recognize other applications where physics
enters their daily lives. To engage students through interactive exploration of the
physics and creation of fun, challenges are one the most important factors can
impact their learning and attitudes towards physics.
International Young Physicists’ Tournament, IYPT, and Persian Young Physicists’
Tournament, PYPT, are new methods in physics education which students know and
understand definitions, terminology, facts, concepts, principles and operations. They
are able to communicate what they know with others and know how to apply what
they have learned to analyze situations and solve problems; and can develop their
ability to evaluate critically the usefulness of various problem-solving approaches.

2. PYPT in Education
To provide a program of educating and supporting teaching assistants, Persian
Young Physicists’ Tournament, PYPT, has been organized to develop assessment
tools to evaluate student progress in problem solving, technical presentation in
English which is not their native language, and team working. The 1st Persian Young
Physicists’ Tournament (PYPT) was in March 2008 which two selected teams
participated in Austria (AYPT) and IYPT to get the first experience from this
attractive challenge. Now students from different high schools in Iran are able to
request entry into PYPT which is carried out in a period determined by the PYPT
Executive Committee (ECO). The best teams challenge in final and receive rewards
but the best students with highest individually scores as the PYPT Regulations are
selected and after education, participate in International Young Physicists’
Tournament, IYPT. The rules for presentation of the results, opposition, reviewing
and judgment by the jury have been fixed in the Regulations of PYPT which is
different from regulations of IYPT in some parts. PYPT Juror are selected from
different universities such as University of Tehran, Amirkabir University of
Technology, Sharif University of Technology, Alzahra University, Islamic Azad
University, Tarbiat Modares Uniersity,…each year
There are three students in a group, from upper secondary schools, up to the age of
18 or 19, which are able to request entry into PYPT. They should work on IYPT
problems which is on the PYPT website too (http://pyptonline.com). There are not
specific solutions for these problems so research and experiment are the main factors
to direct them in finding the best solution.
PYPT workshop is the other educational community which plays an important role in
directing Jurors and students who need to learn more about PYPT regulations.

36
Physics Competitions Vol 13 No 2 2011 & Vol.14 No 1 2012
3. PYPT problems
There are 17 problems should be solved by each team but five problems can be
rejected in PYPT without penalty. In solving these problems students learn how to
research, work in lab, share their findings in their group and compare their results in
experiments with principles and physics’ laws. Some parts of three problems as
follow define how students work to present in an international tournament.
- Magnetic Spring (Problem has been solved by: Reza Montazeri Namin)
Two magnets are arranged on top of each other such that one of them is fixed and
the other one can move vertically. Investigate oscillations of the magnet.

The theory to solve this problem has two main parts: a) To find the force that the
magnets exert to each other in different distances, b) Using the force, to find the
equations of motion of the top magnet. Finding the force may be done using two
main models: The Gilbert’s model, which considers each magnet to be made of two
poles, and every pole acts like an electric pole. The Ampere’s model, which considers
each magnet to be a solenoid, and due to the current in the solenoids, the magnets
exert forces to each other. After finding the force that the magnets exert to each
other, we can find the equations of motion and oscillations of the top magnet. We
know that there are three forces exerted to the top magnet: Gravity, magnetic force
& friction. So we have all the forces that are exerted to the magnet. But again,
because of the complicated form of the magnetic force, the differential equations of
motion will not be solvable, so again we use a numerical method to investigate the
movements.
Similar to the theories, the experiments have two main parts: Checking the force to
decide the correct theoretical model, and investigating the oscillations and comparing
with the theories. The force evaluation was done by adding weights on the top
magnet. By adding weights, the distance between the magnets would change. While
we know the mass of the magnet and the weights, we could find the force that the
magnets exert to each other in different distances (Fig. 1).

Fig. 1. Oscillations of a magnet in different distances

Shrieking rod (Problem has been solved by: Zahra Karimi)


A metal rod is held between two fingers and hit. Investigate how the sound produced
depends on the position of holding and hitting the rod?

Working with the sound waves while there are several types of motion and several
patterns, can be a complicated task. It’s been proved that both longitudinal and
transverse waves can propagate in solids.

37
Physics Competitions Vol 13 No 2 2011 & Vol.14 No 1 2012

In the case of cylindrical metal rod which should be held at a certain position within
its length and be hit, for finding the right patterns of propagation paying attention to
both holding point and hitting type is necessary.
Holding points can be considered as nodes because approximately there is no motion
at these points, these nodes are really critical in understanding the accurate pattern
of longitudinal and transverse waves. A metallic rod is quite similar to a tube which is
open from both ends so when it comes to longitudinal motion only the fundamental
frequency or harmonics which have the same place of nodes with the holding point
can be observed. this means that the longitudinal wave can only be produced by
holding the rod at even multiples of the length (1/2L.1/4L.1/6L,…) while there are
transverse waves caused by vibration from the hitting all the time.(Fig. 2)

Hitting
Hitting
point=end

Hitting
point=1/4L
Amplitude

Frequency

Fig. 2. Transverse waves cause by vibration from the hitting

- Electromagnetic Cannon (Problem has been solved by: Saba Zargham and
Hamid Ghaednia)
A solenoid can be used to fire a small ball. A capacitor is used to energize the
solenoid coil. Build a device with a capacitor charged to a maximum 50V. Investigate
the relevant parameters and maximize the speed of the ball.

38
Physics Competitions Vol 13 No 2 2011 & Vol.14 No 1 2012
As we know, a solenoid consists of helical connected coils. When a current is sent
through the solenoid, it generates a magnetic field, which its magnitude can be
defined by the Bio-Savart law. This magnetic field can exert force on conductive
material. Ferromagnetic materials are made up of many magnetic dipoles.(seen as
current loops) When placed in a magnetic field, these dipoles tend to align with the
field. Due to this alignment a current is produced throughout the matter. This force
can therefore by calculated by the Lorentz force. And it is the same force used to
build the electromagnetic cannon based on the solenoids principle.

The solenoid is wrapped around a diamagnetic hallow cylinder. Depending on the


shape of the coil, a plunger (that consists of two parts, a ferromagnetic part and a
non-ferromagnetic part which is the shooter), can be situated in the solenoid in a way
that can move in and out of the center. The plunger is used to provide mechanical
force which will be used to kick the ball (Fig. 4).

Fig. 4. The plunger used to provide mechanical force in electromagnetic cannon


The final speed of the ball is much related to the force exerted to it by the plunger
which itself is only determined by the force applied to the plunger by the solenoid.
Therefore calculating this force is necessary. To do so, we have first calculated the
force applied to circular-shaped element of the solenoid from a distance r. (the force
along the x axis is the one important for us) and so by integration we’re able to
determine the net force applied to the plunger (Fig. 5).

Fig. 5. Force calculating in Electromagnetic cannon

4]3  _n4DšqMˆf›' < œn%ž


4D  4ž

(The full papers have been published in IYPT 2010-2011 Proceeding ).

4. Statistical Survey

39
Physics Competitions Vol 13 No 2 2011 & Vol.14 No 1 2012

The number of participants in PYPT has increased in recent years in comparison with
last PYPTs (Figs. 6). Students are girls and boys from different schools which during
the period of training, their teachers or team leaders educate them practically. young
students talk about the conceptual approaches which help them in solving problems.
Combination of the knowledge and experiments to motivate students is an important
factor in active learning but to improve it in a high level we need a debating and
asking in a cooperative atmosphere.

Fig. 6. Statistical survey in PYPT

40
Physics Competitions Vol 13 No 2 2011 & Vol.14 No 1 2012
5. Conclusions
In spite of the best efforts of teachers, typical students are also learning that physics
is boring and irrelevant to understanding the world around them. Many college
teachers today want to move past passive learning to active learning, to find better
ways of engaging students in the learning process. But many teachers feel a need for
help in imagining what to do, in or out of class that would constitute a meaningful set
of active learning activities. So we need to change science education to make it more
attractive and relevant for a much larger fraction of the student population than in
the past. PYPT and IYPT is a new method in physics education which lead to greater
retention of knowledge, deeper understanding, and more positive attitudes toward the
subject being taught.

References
1-Izadi D., Torabi A. M., Mahmoudi N.,Izadipanah N.,Eshghi N., “Reports on
Activities in Bssic Sciences in the Last Three Years in Iran”, 4th ICWIP conference,
South Africa, April 5-8, 2011.
2-http://www.pyptonline.com
3-http://www.ayimi.org
4-http://phet.colorado.edu
5-http://www.phys.washington.edu
6-http://groups.physics.umn.edu
7- Carl Wieman and Katherine Perkins, “Transforming Physics Education”, Physics
Today, November 2005, 58(11), p.36
8- Ian D. Beatty Æ William J. Gerace, “Technology-Enhanced Formative Assessment:
A Research-Based Pedagogy for Teaching Science with Classroom Response
Technology”, Published online: 7 January 2009
10- Paosawatyanyong B, P. Wattanakasiwich, “ Implication of physics active-learning
in Asia”, Lat. Am. J. Phys. Educ. Vol. 4, No. 3, Sept. 2010

41
Physics Competitions Vol 13 No 2 2011 & Vol.14 No 1 2012

Operating Temperature and Heat Capacity


of a Light Bulb Filament

An Experimental Problem used in the
German Physics Olympiad
1 Axel Boeltzig, 2 Stefan Petersen
1 Institute of Nuclear and Particle Physics (IKTP), TU Dresden, Germany
2 Leibniz Institute for Science and Mathematics Education (IPN), University of Kiel,
Germany

Abstract
We present an experimental problem used in the German national competition for
the International Physics Olympiad, which, due to its emphasis on accurate data
acquisition and rather unusual ways of evaluating the data, seems very suitable for
testing the experimental abilities and creativity of the students.
The first part of the problem deals with the resistance of a light bulb filament as a
function of temperature. The determination of the filament’s temperature utilizes
features of its black body radiation curve and the wavelength-dependent sensitivity of
a photo diode. The analysis with the given instruments requires clever application of
a logarithmic plot.
In the second task the heat capacity of the light bulb filament is investigated. This can
be done by periodic heating and cooling of the filament. The process is driven by an
appropriate voltage signal from a function generator and monitored using an
oscilloscope. This task can also be used as a single problem, when the result of the
first part is given.

Context
The selection competition for the German IPhO-team, the German Physics Olympiad,
is organized by the Leibniz Institute for Science and Mathematics Education (IPN) at
the University of Kiel and consists of four stages all carried out at a national level. In
the first two stages the students solve sets of given problems at home. In each of the
final two rounds the best participants come together and face two theoretical as well
as two experimental exams, each lasting three to four hours.
The presented experimental problem is in parts based on an idea described by
Kraftmakher (Kraftmakher 2004) and was posed for an exam in the fourth and final
round of the competition in 2011. The 15 participating students had four hours time
to work on it.

Preparation of the students


The second task involves an analogue oscilloscope and a function generator. To
ensure that the participants could focus on physics rather than pure handling of the
devices, an introduction on the equipment used was given two days before the exam.

42
Physics Competitions Vol 13 No 2 2011 & Vol.14 No 1 2012

For a hands-on training the group of students was split into pairs. One student was
asked to set the function generator to given parameters, the other one then asked to
find out these parameters by analyzing the signal with the oscilloscope. The roles
were changed afterwards.
Special attention was paid to aspects that were considered potentially important for
the problem. Some signals were similar to those that could have been used for the
second task.

Problem

Introduction
The predominant process for heat transfer at high temperatures is by radiation. In
this experiment you are asked to examine a filament of a light bulb at high
temperatures.

Material
 Opaque box with four connectors, containing:
- a light bulb with a nominal voltage of 6.0 V (maximum voltage 8.0 V)
- a photo diode (Osram Components BPW 34)
 9 V battery with a resistor of 100 connected in series
 Power supply (variable DC voltage)
 3 multimeters
 Function generator
 Oscilloscope
 Connectors and cables
 (10 ± 0.1) resistor
 4 transparencies with graphs (see problem text and Figure 3)
 Graph paper, ruler, marker, pencil, eraser, adhesive tape
The uncertainty of the multimeters can be estimated as 2 % of the set measuring
range. A list of their inner resistances is available. The ground contacts of
oscilloscope and function generator have the same potential. Avoid short circuits!

Properties and use of the photo diode


The opaque box with the light bulb holds a photo diode. Using this diode, the light
intensity of the bulb can be measured. When a constant voltage is applied to the
diode in reverse direction, it acts as a current source with an output current
proportional to the power of the incident radiation.
However, the constant of proportionality between incident power and output current
depends on the wavelength of the light. For the diode in the box, this relative
sensitivity is shown in Figure 1 (the data was taken from the datasheet mentioned in
the references).

43
Physics Competitions Vol 13 No 2 2011 & Vol.14 No 1 2012

Figure 1: Relative sensitivity of the photo Figure 2: Illustration of the inner box layout
diode as function of the wavelength of the as printed on the box given to the students.
incident light.

Task 1
The graphs on the given transparencies (see Figure 3) show the current of such a
photo diode when it is exposed to the radiation of a black body with different
temperatures T, with no other sources of radiation present and, except for T all
conditions left constant. The graphs differ only in the ways their axes are scaled.
Note that the current signal is given in arbitrary units.

a) Give a brief and qualitative description, how the graphs on the transparencies
can be calculated using the graph from Figure 1.
b) Examine the intensity of the light bulb for different voltages applied to it, in
order to determine the electric resistance of the light bulb at different
temperatures of the filament.
Plot the electric resistance l of the bulb against its temperature 0l and
compare your result to a linear dependence of R Ÿ on 0l .
c) Determine the temperature 0 of the light bulb filament at nominal voltage.

44
Physics Competitions Vol 13 No 2 2011 & Vol.14 No 1 2012

Figure 3: The plots of with different scales as given on the transparencies for the
students. Each plot for the participants was on an A4 transparency.

Task 2
The heat capacity of the filament can be determined with the given devices. To this
end the light bulb can, for instance, be operated by square waves of low frequency
produced by the function generator. The oscilloscope can be used to observe voltage
curves.
Determine the heat capacity of the filament. Name all approximations made, and
the conditions for your result to be valid.

Solution to the experimental problem

Task 1
The fact that the photo diode’s sensitivity depends on the wavelength of the incident
radiation is crucial for this problem. Although the radiation power is proportional to
T ; according to Stefan-Boltzmann law, the sensitivity’s dependence on the
wavelength yields a different relation for the diode signal.
The output current I£ of the photo diode is proportional to the power of the incident
light, weighted by the diode’s spectral sensitivity S λ:
«
4
¦§ ¨ © 0 · S λ · dª

)

45
Physics Competitions Vol 13 No 2 2011 & Vol.14 No 1 2012

The spectral distribution of the filament’s radiation power is given by Planck’s


law. This integral can be evaluated numerically, which yields the given graphs. The
current can only be given in arbitrary units, since the factor of proportionality is not
known in this case.

As stated in the problem text, radiation is the dominant way of losing heat for the
filament. Other means of heat transport can be neglected. This is also true for the
absorption of radiation from the environment.
When the light bulb is operated at a constant voltage ®l with an according current IŸ ,
a thermal equilibrium is quickly established. Thus the bulb’s temperature can be
derived from its power consumption, except for an unknown factor, here called α:
¯
,¦l · ®l  ° · 0l
In addition to the values of ®l and ¦l , the current signal ¦§ of the photo diode can be
measured for each data point.
The expected temperature dependence of ¦§ , given in the graphs of Figure 3, was
discussed above. But since the theory only yields a proportionality, there is another
unknown factor between the measured current and the numerical value given in the
graph, here noted as ±.
So instead of I£ over 0l as given in the graphs, the measurements only yield β · I£
over α · TŸ. However, when plotting log β · I£ ) over log (α · TŸ) one obtains the same
shape of the curve, while the coefficients ° and β are merely reflected as shifts in x -
and y - direction.
Therefore one can match the given logarithmic curve and the measured data by
placing the transparency on top of a graph of the measured values. The same scales
have to be used in both plots and the transparency may not be rotated, of course.
Figure 4 shows the result with the data points shifted onto the given graph from
Figure 3. The error bars are left out for reasons of clarity, which is justified since the
errors become very small at higher temperatures and higher diode currents.
The value of can be derived from the shift in x - direction and subsequently be used
to determine the temperature 0l of the filament from its electrical power
consumption in thermal equilibrium. The uncertainty of ° can be estimated by
variation of the shift. The data aligns well with the full-sized plots for a shift of about
µ2 mm, which is equivalent to a relative uncertainty of about 1.5 % for °. For the
nominal voltage U· one obtains a filament temperature of about T·  2800 µ 709.
The plot of the bulb’s resistance R Ÿ as a function of TŸ in Figure 5 shows very good

linearity, with the resistance of the bulb approximately given by l 0l   22.9 ¹ ·
0l  6.64Ω. Note that the linearity could be checked even without knowledge of °.
The obtained relation l 0l  will prove to be useful in the next task.

46
Physics Competitions Vol 13 No 2 2011 & Vol.14 No 1 2012

Figure 4: Experimental data points shifted Figure 5: The results for and a fitted
onto the logarithmic plot of Figure 3. linear function.

Task 2
The focus for the students’ solutions for this task was on creative application of the
given devices to determine the heat capacity, rather than obtaining a high accuracy.
However, the students were asked to be aware of any assumptions or simplifications
of their method, and to name them.
The heat capacity is relevant for processes out of thermal equilibrium, such as
heating of the filament after switch-on, or cooling after switch-off. Since heating and
cooling in the relevant high temperature range happens on rather short time scales
(typically between 10 and 100 ms), the oscilloscope has to be used to study these
processes. As the given oscilloscope does not provide a storage function, the
observed processes are required to run periodically.
There are different ways to determine the heat capacity. We will present two of them
here.
As suggested in the problem text, one can periodically switch the light bulb on and
off using the function generator. This can be achieved with a square wave of
amplitude J» and an offset of the same value. This means that the bulb is switched on
with a voltage of 2 · J1 for the first part of the cycle and switched off the other time.
The duration and relation of these periods can be changed with different frequencies
and duty cycles of the signal.
To examine the light bulb, its voltage ®l G and current IŸ t need to be measured.
The voltage U) t supplied by the signal generator can directly be displayed on the
oscilloscope. To measure the current, the given resistor can be connected in series to
the bulb, with the voltage U¼ t over the resistor shown on the oscilloscope.
With this setup, the bulb’s voltage and current can easily be derived from the
observed quantities U) t and U¼ t using the following relations:
®) G  ®l G + ® G
® G   · ¦ G   · ¦l G
Note that the electric power ¦l G · ®l G can not be used to determine the filament’s
temperature 0l G here, as no thermal equilibrium is established. However, inverting
R Ÿ (0l  obtained in the first task, the resistance R Ÿ (t) can be used for that purpose.

One way to obtain the heat capacity is to directly determine the heat absorbed by the
filament during a certain time interval of the heating period:
47
Physics Competitions Vol 13 No 2 2011 & Vol.14 No 1 2012

I

½  © ¦l G · ®l G  ° ; · 0l G; 4G
I

The heat capacity then is:


½
¾
0l G   0l G 
This method requires careful and detailed recording of the curves to obtain good
results.
A different and more practical approach is to consider the cooling intervals when the
light bulb is switched off. One can determine the temperatures 0l G 
at the end of a heating period and 0l G  at the beginning of the next one. In
between the filament cools due to radiation:
¾ · 0¿l G  ° ; · 0l G;
After integration this yields:
1 1 °;
  · G  G 
3 · 0l G  = 3 · 0l G = ¾
With different amplitudes, duty cycles and frequencies of the supply voltage, the
initial temperature and the duration of the cooling period can be altered. The formula
above suggests a plot of the left hand side expression as a function of the cooling
time t   t as shown in Figure 6.

Figure 6: Plot to determine the heat capacity from the cooling phase of the
filament. The dashed lines represent a 5 % variation of the fitted curve’s
slope.

Different measured quantities and previous results are used to calculate 0l .


Accordingly the uncertainty of the plotted left hand side expression shown in Figure
6 is determined by many factors. However, the main contribution results from the
two voltage readings on the analogue oscilloscope. The relative uncertainty of the
time difference on the abscissa is negligible.
With the tools available during the exam, the uncertainty of the slope can only be
estimated. A relative uncertainty of 5 %, as indicated in Figure 6, appears to be
justified. For the calculation of C, the uncertainty of discussed before also has to be
considered.

48
Physics Competitions Vol 13 No 2 2011 & Vol.14 No 1 2012
Using the value of determined before and the slope of the linear fit yields the heat
Á
capacity with ¾  2.2 µ 0.2 · 10*À .
¹
In addition to the uncertainty of the result it is also important to recognize the
assumptions used in the presented techniques to determine . For the two methods,
the heat capacity was assumed to be constant in the examined temperature intervals.
This is an approximation, as the specific heat capacity of the material may vary in the
rather large temperature range. The specific heat capacity of tungsten for example
increases by about 20 % in the temperature range from 1500 K to 2500 K according
to (White 1997). Additionally the possibility of other parts than the filament changing
temperature (thus contributing to the heat capacity) was neglected.
Within the scope of this exam, the students could not be expected to test these
assumptions, but should be aware of them.

Conclusions
The two experimental tasks, albeit considered very interesting by the students, proved
to be too extensive for the given amount of time. Even though roughly two thirds of
the 15 candidates were able to achieve reasonable results in the first tasks only two
of them received close to full marks. The second task was more or less successfully
solved by only one of the students. These results can to some extent be attributed to
the lack of time but may also be an effect of the unusual format of the experimental
problem.
Altogether it can be said that the students certainly profited from the intensive
training on the equipment used and that they were rather motivated to work on the
problem. For the purpose of differentiating between the students in the competition
the problem could have been shortened considerably, e.g. to only one of the two
tasks.

Acknowledgements
We gratefully acknowledge the work of Martin Laß and the technical staff at the IPN
who prepared the boxes for the students. Jasmin Andersen, Pascal Cremer, Johannes
Hofmann, Tobias Holder, Martin Krebs and Johann Sawatzky tested the experiment
before the exam and helped grading the students’ results.
The German physics Olympiad is funded by the German Federal Ministry of
Education and Research (BMBF). The selection round was hosted by the school lab
“physik.begreifen” at the Deutsches Elektronen-Synchrotron (DESY).

References
Kraftmakher, Y. (2004). Pulse calorimetry with a light bulb. European Journal of
Physics,25(6):707-715.
White, G. K. and Minges, M. L. (1997). Thermophysical Properties of Some Key
Solids: An Update. International Journal of Thermophysics, 18(5):1269-1327.
Datasheet of photodiode of type Osram Components BPW 34. http://catalog.osram-
os.com/catalogue/catalogue.do;jsessionid=?act=downloadFile&favOid=0200000
30000c226000100b6 (08.11.2011).

49
Physics Competitions Vol 13 No 2 2011 & Vol.14 No 1 2012

A few good orbits

Chiraag Juvekar1, Mehul Jain1 and Aniket Sule2 ,#

1Indian Institute of Technology (Bombay), Mumbai, Maharashtra, India - 400 076.


2Homi Bhabha Centre for Science Education (HBCSE). Mumbai, Maharshtra,India -
400 088.
#Corresponding author: [email protected]

Abstract
We present three novel problems based on Kepler’s laws, which help in bringing out
various facets of orbital dynamics. The problems were designed by the authors for the
training and selection camp of the Indian National Astronomy Olympiad 2008. The
problems require only pre-calculus mathematics and high-school physics, but demand
strong visualization of the scenarios presented. By constructing fun problems
involving elaborate situations like the ones presented here, students are exposed to
practical applications of the Kepler’s Laws like satellite orbits.

1. Introduction
The Indian National Astronomy Olympiad program started in the year 1999. The
Olympiad competitions typically involve pre-university students who compete through
various stages finally leading to selection of national team for respective international
Olympiads. Typical enrollment for the first stage of Astronomy Olympiad in India is
close to 15,000 students. As pre-university curriculum in India does not include
advanced topics in astronomy and astrophysics, initial selection tests focus on the
students’ understanding of physics, mathematics (pre-calculus) and general mental
ability. The last stage of selection is a 3 week summer camp for nationally selected
top 50 students who are exposed to various topics in astronomy and astrophysics,
involving nontrivial physical arguments and pre-calculus mathematics. Designing
problems for the selection tests during the camp is a challenging task as the problems
are expected to be unconventional and yet within boundaries of student’s scholastic
competencies. Over the years a number of fairly sophisticated problems in various
fields of astronomy have been posed in the various levels of the Olympiad program
leading up to the selection of the national team. To level the playing field for all
participants who may have prepared in different ways prior to the selection camp, all
problems in selection tests are newly designed (i.e. none of them are pre-published)
by the Olympiad resource persons, who include teaching faculty as well as past
Olympiad medalists. What follows is an analysis of three problems designed by the
authors in the field of orbital dynamics which were posed in the 10th Indian National
Astronomy Olympiad camp in the year 2008.

50
Physics Competitions Vol 13 No 2 2011 & Vol.14 No 1 2012
2. A Brief Overview of Orbital Mechanics
The basic orbital mechanics is governed by just two equations: Newton’s law of
gravitation and Newton’s second law of motion. The first provides an approximate
expression for the force due to gravitation between two bodies. The second relates
this force to the acceleration of the body enabling us to solve the kinematics of the
bodies involved. Based on these two equations, a complete solution for the two body
problem is well-known as a one-body problem with a reduced mass plus the free
motion of the center of the mass. We can approximate the reduced mass to the mass
of the bigger body (called the central body), if it is many times as massive as the
second (orbiting) body. For this special case, lets us summarize necessary
terminologies, laws etc.
1. Kepler’s Laws:
a. All orbits are conic sections with the central body at one of the foci (let
us call this as the prime focus).
b. The area of the sector, centered at the prime focus, traced by the
orbiting body is proportional to the time taken to cover it.
c. The square of the time period of the orbiting body is proportional to the
cube of the semi-major axis of its elliptical orbit.

2. Peri- and Ap- position: At some point in its orbit the orbiting body must be at
its closest distance to the central body. This is peri-position. Likewise, the
furthest distance is the ap-position. Obviously for unbound obits the ap-
position does not exist (i.e. it is at infinity).
3. True-anomaly(V): The angle subtended by the radius vectors from the prime
focus to the peri-position and the current position of the orbiting body.
The understanding of Kepler’s Laws and teaching the topic in an elegant manner
has been an important topic of discussion amongst physics education community [1],
[2], [3], [4], [5], [6], [7], [9][10].

3. The Solution and the Analysis of the problems


What follows are the actual statements of the problems and their solutions.

3.1. Problem 1 : Celestial Spheres for the Extra-Terrestrial

3.1.1. Problem Statement:


4N1K37-2 is launched from the Earth into a circular geosynchronous orbit and
further it is ensured that the phase of the Earth in the space shuttle’s sky remains
constant The space shuttle’s transmission antenna always points towards the Earth
and the solar panels always face the sun. The solar panels were built with an inherent
gravitational field such that people can stand on them. The space shuttle began its
transmission to the ground station (somewhere on the Equator) when it was in the
Equatorial Plane of the Earth on summer solstice day.

51
Physics Competitions Vol 13 No 2 2011 & Vol.14 No 1 2012

At the beginning of the transmission, an observer Alice who is at the ground station
could see the space shuttle on her zenith and the sun on her western horizon.
Suppose an observer, Bob, standing on the solar panel of the space shuttle is facing
the Earth. Find the constellation in the anti-Earth direction of the observer (on the
space shuttle) after exactly 100 (sidereal) days of transmission.

3.1.2. Motivation:
It is often possible to confuse between a geostationary and a geosynchronous orbit.
In the case of a geo-synchronous satellite all that is required is for the period to be
equal to one day. In other words, for a fixed observer on the ground, it should return
to the same position in the sky at the same time every day. The geo-stationary orbit
is special case of the geo-synchronous orbits, where the orbit is equatorial and
circular. More importantly, the problem demands strong spatio-visual thinking that is
generally neglected in science teaching, especially in India.

3.1.3. Solution:
The problem inherently is very simple. All that is needed is a clear mental picture.
This is the step by step procedure to identify the orbit:
i. Firstly we note that the orbit is a geosynchronous orbit. That means, after 24
hrs the satellite returns to the same position relative to the earth. Thus the
length of the semi-major axis (a) of the orbit is fixed.
ii. The orbit is given to be circular. Hence the eccentricity (e) is also fixed to be 0.
iii. Next we find the inclination (i). It is given that in the satellite sky, Earth has
constant phase. If the Sun-Earth-Satellite angle is θ at a given moment that
after 12 hours it will be (180° -−θ). Thus the phase angle would change unless
of course θ were maintained to be 90°. The orbit which does this is the one
perpendicular to the ecliptic and parallel to the terminator.
iv. Now, we must find the initial position in the orbit. It is given that the first
transmission occurs on summer solstice day. Thus the Earth-Sun vector points
to an ecliptic longitude of 90°. Now the satellite can have 2 positions in this
orbit. One such that the Earth-Satellite vector point to a longitude of 0◦ and the
other such that it points towards 180°. The fact that observer Alice saw the
satellite at her zenith and also saw the Sun in the western sky at the same time
implies that the Earth satellite angle is in fact 180° ecliptic longitude.
v. As for the direction of revolution we could have either but the fact that the
second position is asked after exactly 100 sidereal days renders the point
irrelevant.
Now we note that initially the Anti-Earth direction was the autumnal equinox point.
After 100 days which correspond to approximately 3 and months, the constellation
will shift by three zodiac constellations. Initially the anti-Earth side was Virgo and
finally it turns out to be Sagittarius. (Note: Rough Calculations show that the ecliptic
longitude turns out to be 278)

52
Physics Competitions Vol 13 No 2 2011 & Vol.14 No 1 2012
3.1.4. Remarks:

Although the problem appeared to be highly calculation oriented, no actual


calculation was required to arrive at the first order solution. All that is needed is a
step by step analysis aided by strong visualization and the realization that the
geosynchronous nature of the orbit is the key input.

3.2. Problem 2 : A Tale of two Phases

3.2.1. Problem Statement:


In the year 2021 the ISRO’s home-bred space shuttle identified by the code MJ-3105
is planned to dock with our space observatory 4N1K37 which orbits the earth at 600
km in circular orbit. Imagine, that due to some slight calculation errors the MJ lands
in the correct orbit but at a wrong phase such that the distance between MJ and the
observatory is 916 km and the observatory is ahead of MJ in its orbit. (Assume the
orbit of 4N1K37 to be a non-precessing circle). As a corrective measure an
appropriate elliptical orbit is suggested. If MJ-3105 is constrained by the fact that it
can only apply thrust in a direction tangential to its orbit,
i. What possible orbits can it be put into such that after one complete periodof
MJ in its new orbit, its location and the observatory’s location will turn out to
be the same? Draw a neat diagram depicting the same. (Note: After one
complete revolution the corresponding thrust in the opposite direction will
restore the circular orbit, thus completing this phase correction sequence.)
Figure for problem 2
ii. Find the minimum thrust required for this complete maneuver and also find the
corresponding orbit.
iii. If the initial phase angle difference was 30◦ instead, what would be the
minimum thrust?

3.2.2. Motivation:
Inorder to change the phase of an object in orbit around a central body one needs to
exert some sort of external force on it. This force will change the momentum of the
body at that given point in the orbit. Now as a result of this new velocity the orbit
itself changes defeating the whole purpose of the initial momentum. The solution to
this Catch-22 situation is the selection of certain specific orbits set out in this
problem.

3.2.3. Solution:
Consider the figure below. Now we know that the initial straight line distance
between 4N1K37 and MJ-3105, with both in same circular orbit is 916km.

53
Physics Competitions Vol 13 No 2 2011 & Vol.14 No 1 2012

We can calculate the chord length as,


Y
4  2 sin ž⁄2 i.e. ž  2%_@*  
'

Now let TMJ be the time taken by MJ to complete one orbit. We know by Kepler’s
third law that inner orbits have smaller time period and that the outer orbits will have
a larger time period. Consider the orbits shown in the figure1. For 4N1K37 and MJ
to come to same position, when MJ is in an outer orbit, 4N1K37 must cover an
integral number of orbits less an angle θ in one orbital period of MJ. However when
MJ is in an inner orbit, it must cover an integral number of orbits in time required for
4N1K37 to cover an angle 2−θ in its orbit. Hence the governing conditions for
circular orbit of 4N1K37 are
Figure Different maneuvering orbits discussed in problem 2.
2 @ + 1O  ž 2O  ž
0 Á,uMI›' x y 0; @Ã0 Á,Rˆˆ›' Ä  ‚ 0;
2O 2O

By Kepler’s Third law,



o= Á ;= 0 Á Œ
 o Á  ;  ‚
0 Á 0; 0;

Thus we get two cases for inner and outer orbits,


 
ž Œ 1 4 Œ
o Á,uMI›'  ; @ + 1  ‚  ; @ + 1  %_@* ‚
2O O 2/
 

1 ž Œ 1 1 *
4 Œ
o Á,Rˆˆ›'  ; x 1  ‚y ; Å 1  %_@ ‚Æ .
@ 2O @ O 2/
For an elliptical orbit, using energy conservation, velocity at a distance r from the
focus is given by,

2 1
  ƒÇ-   ‚.
/ o

54
Physics Competitions Vol 13 No 2 2011 & Vol.14 No 1 2012
Hence thrust per unit mass is given by,
Í
Œ
1 2 1 Ç- ÌÉ 1 4 Ð
È  ÉÇ- Šƒ  ƒ  ‹ ÈuMI›'  ƒ Ë 2  Å@ + 1  %_@*  ‚Æ  1Ï ÈRˆˆ›'
/ / o ; O 2/
Ê Î
Í
Ç- 1 1 4 Œ
ƒ Ñ1  ƒ2  1  %_@* ‚ Ò.
; @ O 2/

Now note, for the outer orbits as the order of the orbit (n) increases, the thrust must
increase as for the same perigee point the semi-major axis is increases. Similarly for
the inner orbits as n increases for the same apogee point the semi-major axis is
decreasing. Hence both the velocity difference and the thrust are more. Hence the
order of both orbits should be 1. Explicitly calculating ratio of thrust to initial
momentum values we see,

ÈuMI›',:?Ó(  0.1681 , ÈRˆˆ›',:?Ó(  0.0071


ÈuMI›',=)°  0.1627 , ÈRˆˆ›',=)°  0.0303

Thus it seems as though, both the inner orbits are favoured. As a final consistency
check we calculate the ratio of the perigee distance to the radius of the earth. Note
for inner orbits the current point is at apogee.

1 4 Œ ;
/š›'Rf››  2o  /Qšuf›› S  Š2 1  %_@* ‚  1‹ S  1.062S=)°  0.970
O 2/ › :?Ó(

Thus, in the second case, perigee turns out to be inside the Earth. Hence, even
though the inner orbit is favoured due to low thrust, only the outer orbit can be
chosen. Initial velocity is the circular velocity i.e.

Ç-
ƒ · 7.561 #$⁄%
;

Hence, we have ÈRˆˆ›',:?Ó(  53.6 $⁄% and ÈuMI›',=)°  1230 $⁄%

3.2.4. Remarks:
The key ideas to learn from the problem are:
i. If a body applies tangential force the initial point is constrained to be either the
peri-point or ap-point.
ii. In order to change the phases of the orbit we can select a few quantized orbits
such that the satellite can make up for the lost phase by going around once in
a new orbit.
iii. For any orbital maneuver it is imperative to check whether the modified orbit

55
Physics Competitions Vol 13 No 2 2011 & Vol.14 No 1 2012

collides with the central body. This check must be externally imposed and
cannot be guessed from the initial conditions.

3.3. Problem 3: The Great Escape

3.3.1. Problem Statement:


An earth-like planet ‘Echor’, resides in a stellar system at approximately 10 A. U.
from its parent star, which is similar to the Sun. The planet has no natural satellite
and other planets in that system are far from this planet. The Echorans have decided
to launch their first interstellar mission to colonize other habitable planets in other
star systems. The starship ‘Chabya’ designed for the purpose has the following
dimensions. It consists of two spherical sub-ships (H and B), approximately 20km in
diameter each, attached to one another by a small tether in the center. The density of
the spaceships is the same as that of the planet i.e. approximately 5.5 g/cc. The first
step to get the massive Chabya to the closest star is to get it out of the gravitational
influence of Echor. For this purpose the ship is firstly put up in a circular orbit about
Echor with radius equal to12,960,500km. The ship is oriented such that the centers
of the two sub ships and the center of Echor are always collinear and Chabya-B
always lies on the inside. It is believed that when an appropriate retarding impulse is
provided, the spacecraft will go into a new orbit and at somepoint, Chabya-H will be
able to escape outward leaving Chabya-B behind. Find the minimum retarding
impulse to achieve this. (Note: The breaking strength of the tether is 7x1012 N.)

3.3.2. Motivation:
The main idea behind this problem is that for a system of rigid bodies with no relative
rotation, the velocity of each body is same as that of the center of mass. However,
when the individual parts separate, they continue to have the same initial velocity but
may move differently based on the differential forces acting on them. In addition, for
a given velocity, the orbit depends on the gravitational potential at that point. Thus,
for a small change in initial position, the final positions of the bodies may be
remarkably different. Another key idea involved is that parameters of the problem are
such that mutual gravitational force between the bodies is not to be ignored.

3.3.3. Solution:
Firstly, consider the two satellite system. The forces acting on each component are a
net inward force due to both tension and gravity. In the frame of the center of mass
of the system, there is a net outward force on each body. And a differential force due
to gravity. Consider the force equation for Chabya-H,
]՛ˆI'RÖMfQq  ]f'QLRIQIRuˆQq + 0ˆ›I

56
Physics Competitions Vol 13 No 2 2011 & Vol.14 No 1 2012
For Chabya-B,
]՛ˆI'RÖMfQq  ]f'QLRIQIRuˆQq  0ˆ›I

Now the tension is a combination of tether tension and gravitational interaction.


Ç$ 4ÇO  / ; S
0ˆ›I  0f'QLRIQIRuˆQq + 0I›Iכ'  + 0I›Iכ'  + 0I›Iכ'
2/ 9
Where, r is the radius of each sub-part.
In this orbit, we have 0ˆ›I  1.41 · 10À Ø.
Now note that the mutual gravitational force between the Chabya-B and the
Chabya-H in the initial orbit is much larger than the limiting breaking strength of the
tether. Almost the entire contribution to the Tnet comes from the mutual
gravitational force between the two components. Hence the tether cannot
automatically break in this orbit. To separate the two parts automatically, one must
move the satellite to a point where their mutual gravitational attraction can be
overcome; or in other words move it near the Roche Limit.
Now the Roche Limit is given by,

2S Œ 
4 uÕכ  šqQˆ›I  ‚  šqQˆ›I 2Œ .
S(
We give a retarding impulse to the satellite such that it goes in an elliptic orbit
with the initial orbit position as the ap-point and the Roche limit distance as the peri-
point. We need not go any closer, as when the tether will be at the Roche limit,
Chabya-B will be inside the Roche sphere and Chabya-H will be just outside and the
tension in the tether will just exceed the breaking strength. On separation of the
tether, both the parts will continue to have the center of mass velocity. As such
Chabya-H which is outside may overcome the gravitational potential and escape
whereas Chabya-B which is inside will fall inwards.
Once again, we use the expression for velocity of any orbiting body in an elliptic
orbit

2 1
  ƒÇ-   ‚ .
/ o

Hence to make the Roche limit distance as the peri-point, the velocity at the ap-
point should be,

2 1 2Ç- š›'R 2Ç- 4 uÕכ


Qšu  ƒÇ-   ‚ƒ x y Qšu  ƒ  ‚
o 1 + E o Qšu + š›'R Qšu ÕR'ÕMqQ' + 4 uÕכ ÕR'ÕMqQ'

For the initial circular orbit,

Ç-
ÕR'ÕMqQ'  ƒ
ÕR'ÕMqQ'

57
Physics Competitions Vol 13 No 2 2011 & Vol.14 No 1 2012

Hence we have,

Ç- 2Ç- 4 uÕכ
∆  ÕR'ÕMqQ'  Qšu  ƒ ƒ  ‚
ÕR'ÕMqQ' ÕR'ÕMqQ' + 4 uÕכ ÕR'ÕMqQ'
¯
Ç- 2Œ šqQˆ›I
ƒ Ñ1  É  Ò.
ÕR'ÕMqQ' ÕR'ÕMqQ' + 2Œ šqQˆ›I

Thus the change in momentum is given by,


¯
Ç- 2Œ
∆b  $ÚQI›qqRI› ƒ Ì1  É 
Ð.
ÕR'ÕMqQ' Û|ÜÛÝ{ÞÜ
+2Œ
Ê ß{Þàáâ Î
Noting that it is an ‘Earth-like’ planet, we use values of the Earth mass and the Earth
radius. The numerical value of this change is 3.2×1019 kg m/s.

3.3.4. Remarks:
(i) Although the gravitational force of everyday bodies is negligible, one must
consider it when body sizes are substantially large and they are placed in a
weak gravitational potential.
(ii) The reason why Chabya-H leaves is not clear, immediately, however careful
analysis shows us that if the split between Chabya B and H occurs at the
right spot then the two will separate and the initial momentum will carry
one part out but the other will fall in.

4. Summary
The above problems are meant to give students a flavor of the vast and exciting field
of Orbital Dynamics. The attempt was to show that in orbital mechanics, problems
which look daunting at the first-sight can be solved with nothing more than essential
simple physical arguments and knowledge of pre-college mathematics. It must be
noted that each of these problems stimulates the student to think in an ’out-of-the-
box’ manner, and emphasizes concepts rather than mechanical calculations. Exposure
to such problems will hopefully attract meritorious students to the wonderful world of
Astronomy.

Acknowledgement
The authors would like to thank all the persons associated with the Indian National
Astronomy Olympiad Programme.

58
Physics Competitions Vol 13 No 2 2011 & Vol.14 No 1 2012
References
[1] Kenneth J. Adney, “Applying Kepler's third law”, Phys.Teach., 25, 493 (1987)
[2] Manfred Bucher, “Kepler's third law: Equal volumes in equal times”, Phys. Teach.
36, 212 (1998)
[3] Harold Cohen, “Testing Kepler's laws of planetary motion”, Phys. Teach. 36, 40
(1998)
[4] Mario Iona, “Remember properties of conic sections”, Phys. Teach. 39, 20
(2001)
[5] Juan Lin, “A demonstration of Kepler's third law”, Phys. Teach., 31, 122 (1993)
[6] James L. Mariner & Jack K. Horner, “Kepler's second law and synodic period”,
Phys. Teach. 19, 116 (1981)
[7] Ellis D. Noll, “Kepler's third law for elliptical orbits”, Phys. Teach. 34, 42 (1996)
[8] A E Roy and D Clarke, “Astronomy: Principles and Practices”, 4th ed., (Institute
of Physics Publishing, 2003)
[9] Michael J. Ruiz, “Kepler’s Third Law Without a Calculator”, Phys. Teach. 42,
530 (2004)
[10] Stephen B. Turcotte, “Orbital Timing for a Mission to Mars”, Phys. Teach. 43,
293 (2005)

59

You might also like